Modeling a Damped Spring System with Differential Equations

Click For Summary
The discussion revolves around modeling a damped spring system using differential equations. An 8-pound mass attached to a spring stretches it to 6.6 feet, and the damping constant is given as one. The equation of motion is derived, leading to a form of y'' + 4y' + 80/3 y = 0, but there are concerns about the interpretation of weight versus mass and the meaning of the damping constant. Clarification is provided that 8 pounds is indeed weight, and the mass can be calculated by dividing by the acceleration due to gravity. The damping force is understood to be proportional to speed and acts in the opposite direction of motion.
joker2014
Messages
21
Reaction score
0

Homework Statement



After a mass weighing 8 pounds is attached to a 5-foot spring, the spring measures 6.6 feet. The entire system is placed in a medium that offers a damping constant of one. Find the equation of motion if the mass is initially released from a point 6 inches below the equilibrium point with a upward velocity of 1 ft/sec.

Homework Equations


F=kx
my''+cy'+ky=0

The Attempt at a Solution


I got k=8/(6.6-5) = 8/1.2
c= 1 as given ?

setting up the eqn I got 1/4 y'' + 1y' + 8/1.2 y = 0
or y'' +4y' + 80/3 y = 0

is this actually right? I tried 3 times and I keep getting this, i don't know i feel suspicious ?!
 
Physics news on Phys.org
"8 pounds" is weight, or force, not mass. What does "damping constant" mean? What are its units and how does it give force?
 
HallsofIvy said:
"8 pounds" is weight, or force, not mass. What does "damping constant" mean? What are its units and how does it give force?
of course 8 lbs is weight, dividing by 32 gives me .25 which is my mass! damping i believe is given in the problem "damping constant of one" .. otherwise if not given i would find it by sqrt of 4*m*k
 
"Damping force" is always opposite to the motion and is, approximately, proportional to the speed or the speed squared. I presume here you are told that it is proportional to speed. But because it is opposite to speed, the force must be -kv.
 
and of course my iniital conditions would be y(0)=-6 y'(0)=1
 
Question: A clock's minute hand has length 4 and its hour hand has length 3. What is the distance between the tips at the moment when it is increasing most rapidly?(Putnam Exam Question) Answer: Making assumption that both the hands moves at constant angular velocities, the answer is ## \sqrt{7} .## But don't you think this assumption is somewhat doubtful and wrong?

Similar threads

Replies
7
Views
2K
  • · Replies 7 ·
Replies
7
Views
2K
  • · Replies 27 ·
Replies
27
Views
2K
  • · Replies 9 ·
Replies
9
Views
2K
  • · Replies 9 ·
Replies
9
Views
3K
  • · Replies 7 ·
Replies
7
Views
2K
Replies
10
Views
2K
  • · Replies 12 ·
Replies
12
Views
2K
Replies
3
Views
2K
  • · Replies 1 ·
Replies
1
Views
2K